Verbal Section - GMAT; Manhattan Prep

Réussis tes devoirs et examens dès maintenant avec Quizwiz!

Circle the pronoun, underline the antecedent, and correct the sentence if needed. Most European countries—including those of Bulgaria and Romania—have joined the European Union; Norway and Switzerland, however, have steadfastly refused to do it.

"those of" is redundant, it -> so

Underline the idiom, then evaluate as RIGHT, SUSPECT, or WRONG: (a) The ring-tailed squirrel is more adept at surviving harsh winter conditions as its cousin, the golden-mantled squirrel. (b) The ring-tailed squirrel is more adept at surviving harsh winter conditions rather than its cousin, the golden-mantled squirrel. (c) The ring-tailed squirrel is more adept at surviving harsh winter conditions instead of its cousin, the golden-mantled squirrel. (d) The ring-tailed squirrel is more adept at surviving harsh winter conditions than its cousin, the golden-mantled squirrel.

a. wrong b. wrong c. wrong d. right

Fix the sentence, (a) circle the parallelism markers, and (b) place [square brackets] around each set of parallel elements. Tobacco companies, *shaken by a string* of legal setbacks in the United States, *but* which retain strong growth prospects in the developing world, face an uncertain future.

a."," but b. [WHICH have been shaken by a string of legal setbacks in the United States, but WHICH retain strong growth prospects in the developing world]

Circle the pronoun, underline the antecedent, and correct any errors: The Smiths avoid the Browns because they dislike their children.

ambiguous One possible version: The smiths avoid the Browns because the Browns dislike the Smiths' children

Fill in the blank space with "like" or "as" Eyewitnesses describe the missing passenger _______ a lawyer in his late forties.

as

Fill in the blank space with "like" or "as" Having passed the state bar exam, she is licensed to work a _______ lawyer in Illinois.

as

Fill in the blank space with "like" or "as" Mrs. Jones watched _______ a child played with a stick.

as

Fill in the blank space with "like" or "as" _______ a child has been injured, we must stop the party and call an ambulance.

as

Fill in the blank space with "like" or "as" _______ a child, Rebecca lived in Bristol.

as

Underline comparison, write correct or correct the sentence: I scored three goals in yesterday's game, as did Suzanne.

as CORRECT

Underline comparison, write correct or correct the sentence: A leopard cannot run as fast as a cheetah.

as fast as CORRECT

Underline comparison, write correct or correct the sentence: A leopard cannot catch a wildebeest as fast as a cheetah.

as fast as cannot catch a wildebeest as fast as -> cannot catch a wildebeest that runs as fast as

Underline comparison, write correct or correct the sentence: A leopard's skill in catching a wildebeest is as impressive as a cheetah.

as impressive as cheetah -> cheetah's

Correct the error in the underlined section (if there is one.) The *athlete's wearing* the Brand X logo is a famous Olympian; *his swimming* has led to a lucrative endorsement contract.

athlete wearing

Correct the error in the underlined section (if there is one.) New regulations require that every cyclist in the Tour de France *has to be tested* for performance-enhancing substances.

be tested

Circle the pronoun, underline the antecedent, and correct any errors: Some people believe that the benefits of a healthy diet outweigh that of regular exercise.

benefits--those that -> those

Fix the circled section, do not change anything that is not circled: The negotiations *between* the company, the union, and the city government were initially contentious but *ultimately* amicable.

between -> among

Fix the circled section, do not change anything that is not circled: *Between* 1998 and 2003, there was heavy fighting in Parthia *between* numerous armed factions *yet* this conflict, so much more complicated than a conventional war (between) two states, involved no *less* than eight countries and twenty-five militias.

between₂ -> among, less -> fewer

Fix the underlined section: In the Fischer—Tropsch process, which *developed* in Germany by Franz Fischer and Hans Tropsch, coal *is converted* into a liquid fuel similar to petroleum.

developed -> was developed

Correct the error in the underlined section (if there is one.) Although Bernard normally *is eating* inexpensive foods, and indeed *is eating* a hot dog right now, he *is eating* lobster and steak at tomorrow's party.

eats--is eating--will eat

Underline comparison, write correct or correct the sentence: The clothes looked more appealing inside the store than on the racks outside.

more appealing CORRECT

Underline comparison, write correct or correct the sentence: The clothes inside the store looked more appealing than on the racks outside.

more appealing than the clothes inside the store looked more appealing than -> the clothes looked more appealing inside the store than

Underline comparison, write correct or correct the sentence: Thomas is more interested in video games than his girlfriend.

more interested than more interested in video games than his girlfriend -> more interested in video games than his girlfriend is

Fix the circled section, do not change anything that is not circled: Most legislators—including *much* in the governor's own party—realize that the governor's budget would imperil the state's finances*, nonetheless,* the budget is likely to be approved*, because* few legislators want to anger voters by cutting spending or raising taxes.

much -> many

Circle the pronoun, underline the antecedent, and correct any errors: Oil traders have profited handsomely from the recent increase in its price.

none CORRECT

Fix the circled section, do not change anything that is not circled: Jim is trying to reduce the *number* of soda that he drinks*,* at last night's party, however, his resolve to drink *fewer* soda was sorely tested*,* he found himself quaffing *many* of sodas.

number -> amount, fewer -> less, "," -> and

Circle the pronoun, underline the antecedent, and correct the sentence if needed. The police have significantly reduced violent crime and are pleased with them for doing it.

police--themselves and are pleased with them -> and are pleased with themselves

Fix the underlined section: *The dealer was asked to sell a painting by Picasso.*

-> Picasso asked the dealer to sell a painting OR The dealer was asked to sell a Picasso painting.

Using the underlined modifiers, (1) identify the word that it modifies and (2) indicate whether the modifier is correct. If incorrect fix the error. The houses *on Canal Street*, *of which many had been damaged in the storm*, looked *abandoned*.

1. houses, houses, houses 2. of which many -> many of which

Using the underlined modifiers, (1) identify the word that it modifies and (2) indicate whether the modifier is correct. If incorrect fix the error. The principal tried to calm the *worried* students' parents, as a result of those *students* fighting at the pep rally.

1. parents, fighting 2. students' parents, as a result of those student fighting -> worried parents of the students who had fought

Fix the sentence: After the test format was changed, scores subsequently dropped by more than a 25% decrease.

After the test format was changed, scores dropped by more than 25%.

Circle the pronoun, underline the antecedent, and correct any errors: All students need his or her own copy of the textbook in order to take the class.

All students--their his or her -> their

Fix the sentence: Though canals have experienced a severe decline in barge traffic over the past several decades, yet with the rise in fuel costs, "shipping" by again become an important means of transporting goods within the country.

Canals have experienced a severe decline in barge traffic over the past several decades, yet with the rise in fuel costs, "shipping" by actual ships may once again become an important means of transporting goods within the country.

Circle the pronoun, underline the antecedent, and correct the sentence if needed. Caroline receives email from friends who she knows well, from acquaintances who's names are only vaguely familiar, and from strangers about who she knows nothing at all.

Caroline--she, friends--whom, acquaintances--whose, strangers--whom, she who₁ -> whom, who₂ -> whom, who's -> whose

To increase the productivity of the country's workforce, the government should introduce new food guidelines that recommend a vegetarian diet. A study of thousands of men and women revealed that those who stick to a vegetarian diet have IQs that are approximately five points higher than those who regularly eat meat. The vegetarians were also more likely to have earned advanced degrees and hold high-paying jobs.

Conclusion - Premise - Premise

Identify the role of each sentence or major piece of information: Editorial: to stem the influx of illegal immigrants, the government is planning to construct a wall along our entire border with Country Y. This wall, however, will do little to actually reduce the number of illegal immigrants. Because few economic opportunities exist in Country Y, individuals will simply develop other creative ways to enter out nation.

Counterpoint - Conclusion - Premise

Identify the role of each sentence or major piece of information: Some critics have argued that the price of food and drink at Ultralux, a restaurant, is too high for the quality offered. However, Ultralux features a beautiful interior and comfortable seating, and research has shown that consumers actually enjoy food and drink more in such a setting, even when the food and drink are of comparable quality to the food and drink served elsewhere. Thus, the food and drink at Ultralux are reasonably priced.

Counterpoint - Premise - Premise - Conclusion

Most doctors recommend consuming alcohol only in moderation, since the excessive intake of alcohol has been linked to several diseases of the liver. Drinking alcohol is no more dangerous for the liver, however, than abstaining from alcohol entirely. Last year, more nondrinkers that drinkers were diagnosed with liver failure.

Counterpremise - Conclusion - Premise

Identify the role of each sentence or major piece of information: The cutback in physical education is the primary contributing factor to North High School's increasing failure rate on the year-end physical fitness examination. Last year, when students participated in gym class on a daily basis, 85% of the school's seniors passed the exam. This year, students had gym class twice weekly, and only 70% of seniors passed the test. Clearly, fewer sessions of gym class lead to reduced fitness.

Final Conclusion - Premise - Premise - Intermediate Conclusion

Underline comparison, write correct or correct the sentence: In contrast to the trapeze artists, who fumbled their routine, the antics of the circus clowns kept the audience entertained for hours.

In contrast to the antics of the circus clowns -> thie circus cllowns kept the audience entertained with their antics

Circle the pronoun, underline the antecedent, and correct the sentence if needed. It would hardly be fair for the meatpacking industry to blame regulators for the harm that it has inflicted upon itself in the sub-prime meat sector.

It--blame regulators, it--meatpacking industry, itself CORRECT

Identify the role of each sentence or major piece of information: Tay-Sachs disease, a usually fatal genetic condition caused by the buildup of gangliocides in nerve cells, occurs more frequently among Ashkenazi Jews that among the general population. The age of onset is typically six months and generally results in death by the age of four.

Premise - Premise

Identify the role of each sentence or major piece of information: The Chinese white dolphin is a territorial animal that rarely strays far from its habitat in the Pearl River Delta. In recent years, increasing industrial and agricultural runoff to the delta's waters has caused many white dolphins to perish before they reach breeding age. Unless legislation is enacted to ensure there is no further decline in the delta's water quality, the Chinese white dolphin will become extict.

Premise - Premise - Conclusion

Fix the sentence, (a) circle the parallelism markers, and (b) place [square brackets] around each set of parallel elements. The experiences we have *in childhood* influence our behavior as adults.

a. b. AS CHILDREN, AS ADULTS

Fix the sentence, (a) circle the parallelism markers, and (b) place [square brackets] around each set of parallel elements. We were dismayed to learn that our neighbors were untidy, disagreeable, and *they were uninterested to make* new friends.

a. ",", "," and b. [UNTIDY , DISAGREEABLE, and UNINTERESTED in making new friends]

Fix the sentence, (a) circle the parallelism markers, and (b) place [square brackets] around each set of parallel elements. Voters want to elect a president who genuinely cares about health care, the *environment the travails of ordinary men and women has the experience wisdom,* and strength of character required for the job.

a. ",", "," and, and, "," and, ",", "," and b. [WHO genuinely cares about {health care, the environment, and the travails of ordinary [men and women], and women}], and [WHO has the {experience, wisdom, and strength of character} required for the job]

Fix the sentence, (a) circle the parallelism markers, and (b) place [square brackets] around each set of parallel elements. Although we were sitting in the bleachers, the baseball game was as exciting to us *as the people* sitting behind home plate.

a. Although, as, as b. [as exciting TO us as TO the people]

For each of the underlined modifiers, (a) identify the word or words, if any, that it modifies, and (b) indicate whether the modifier is correct. If the modifier is incorrect, suggest a way to correct the error. Angola, *which was ravaged by civil war for many years after it gained independence from Portugal*, *which is now one of Africa's success stories*, has an economy that grew by 21% last year, *where parliamentary elections are to be held later this week*.

a. Angola, Portugal, Angola b. correct, which -> Angola is now, where parliamentary -> and parliamentary

For each of the underlined modifiers, (a) identify the word or words, if any, that it modifies, and (b) indicate whether the modifier is correct. If the modifier is incorrect, suggest a way to correct the error. *Unaccustomed to the rigors of college life*, James's grades dropped.

a. James's grades b. James's grades dropped -> James allowed his grades to drop.

For each of the underlined modifiers, (a) identify the word or words, if any, that it modifies, and (b) indicate whether the modifier is correct. If the modifier is incorrect, suggest a way to correct the error. Mary buys cookies made with SugarFree, *an artificial sweetener, which tastes as sweet as the corn syrup that her brother loves* but *having fewer calories than in an equivalent amount of corn syrup*.

a. SugerFree, SugarFree, SugarFree b. correct, correct, having -> which has

For each of the underlined modifiers, (a) identify the word or words, if any, that it modifies, and (b) indicate whether the modifier is correct. If the modifier is incorrect, suggest a way to correct the error. Last night our air conditioner broke, *which caused great consternation*.

a. air conditioner b. which caused -> causing

Fix the sentence, (a) circle the parallelism markers, and (b) place [square brackets] around each set of parallel elements. The joint business venture will increase employee satisfaction and *be improving relations* between upper management and staff.

a. and b. INCREASE employee satisfaction and IMPROVE relations between upper management and staff].

Fix the sentence, (a) circle the parallelism markers, and (b) place [square brackets] around each set of parallel elements. The blizzard deposited more than a foot of snow on the train tracks, *prompted the transit authority to shut down service temporarily, and* causing discontent among commuters who were left stranded for hours.

a. and b. [PROMPTING the transit authority to shut down service temporarily and CAUSING discontent among commuters who were left stranded for hours]

Fix the sentence, (a) circle the parallelism markers, and (b) place [square brackets] around each set of parallel elements. The consultant is looking for a café *where there are comfortable chairs* and that provides free internet access.

a. and b. [THAT has comfortable chairs and THAT provides free internet access]

Using the corrected sentence, (a) circle the parallelism markers, and (b) place [square brackets] around each set of parallel elements. The network security team is responsible for detecting new viruses and the creation of software patches to block those viruses.

a. and b. [the DETECTION of new viruses and the CREATION of software patches to block those viruses].

Using the corrected sentence, (a)circle the parallelism markers, and (b) place [square brackets] around each set of parallel elements. Most employers agree that how a candidate dresses for a job interview and even the way he positions himself in his seat leave a lasting impression.

a. and b. [HOW a candidate dresses for a job interview and even HOW he positions himself in his seat]

(a) circle the verb, (b) underline the subject, (c) determine whether the subject and the verb make sense together and (d) determine whether the subject agrees in number with the verb. Just around the corner is a fantastic bakery and a small supermarket.

a. are b. bakery AND supermarket c. yes d. no

(a) circle the verb, (b) underline the subject, (c) determine whether the subject and the verb make sense together and (d) determine whether the subject agrees in number with the verb. There is, according to my doctor, many courses of treatment available to me.

a. are b. many courses c. yes d. no

Fix the sentence, (a) circle the parallelism markers, and (b) place [square brackets] around each set of parallel elements. Researchers have found a correlation between *exercise* and earning good grades.

a. between, and b. [between EXERCISING and EARNING good grades]

Fix the sentence, (a) circle the parallelism markers, and (b) place [square brackets] around each set of parallel elements. The band chosen for the annual spring concert appealed to both *the student body as well as to the administration.*

a. both, and b. [both the student BODY and the ADMINISTRATION]

(a) circle the verb, (b) underline the subject, (c) determine whether the subject and the verb make sense together, and (d) determine whether the subject agrees in number with the verb. Tired of practicing, the orchestra decide to walk out on its astonished conductor.

a. decides b. the orchestra c. yes d. no decide -> decides

For each of the underlined modifiers, (a) identify the word or words, if any, that it modifies, and (b) indicate whether the modifier is correct. If the modifier is incorrect, suggest a way to correct the error. Regina returned the dress to the store, *which was torn at one of the seams*.

a. dress b. Regina returned the dress, which was torn at one of the seams, to the store.

For each of the underlined modifiers, (a) identify the word or words, if any, that it modifies, and (b) indicate whether the modifier is correct. If the modifier is incorrect, suggest a way to correct the error. Of all the earthquakes in European history, the earthquake, *which destroyed Lisbon in 1755*, is perhaps the most famous.

a. earthquake b. which -> that and remove last comma

Fix the sentence, (a) circle the parallelism markers, and (b) place [square brackets] around each set of parallel elements. The museum displays the work of a wide variety of artists, from those who are *world-renowned to who* are virtually unknown.

a. from, to b. [from THOSE WHO are world-renowned to THOSE WHO are virtually unknown]

(a) circle the verb, (b) underline the subject, (c) determine whether the subject and the verb make sense together, and (d) determine whether the subject agrees in number with the verb. Whoever rented these movies has to take them back before midnight.

a. has b. whoever rented these movies c. yes d. yes

(a) circle the verb, (b) underline the subject, (c) determine whether the subject and the verb make sense together and (d) determine whether the subject agrees in number with the verb. A number of players on the team have improved since last season.

a. have improved b. number c. yes d. yes

(a) circle the verb, (b) underline the subject, (c) determine whether the subject and the verb make sense together, and (d) determine whether the subject agrees in number with the verb. A congressional majority is opposed to the current policy.

a. is b. A congressional majority c. yes d. yes

(a) circle the verb, (b) underline the subject, (c) determine whether the subject and the verb make sense together, and (d) determine whether the subject agrees in number with the verb. Planting all these seeds is more involved than I thought.

a. is b. Planting all these seeds c. yes d. yes

(a) circle the verb, (b) underline the subject, (c) determine whether the subject and the verb make sense together and (d) determine whether the subject agrees in number with the verb. Jack, along with-some-of-his closest friends, is sharing a limo to the prom.

a. is sharing b. Jack c. yes d. yes

Using the corrected sentence, (a) circle the parallelism markers, and (b) place [square brackets] around each set of parallel elements. Dr. King's "Letter from a Birmingham Jail" is a condemning of racial injustice and a calling for nonviolent resistance to that injustice.

a. is, and b. [Dr. King's "Letter from a Birmingham Jail" is {a CONDEMNATION of racial injustice and a CALL for nonviolent resistance to that injustice}]

(a) circle the verb, (b) underline the subject, (c) determine whether the subject and the verb make sense together, and (d) determine whether the subject agrees in number with the verb. Although progress is still difficult to measure, the researchers have found that the benefit of applying interdisciplinary approaches and of fostering cooperation across multiple teams and divisions outweigh any potential cost.

a. is, have found, outweigh b. progress, the researchers, the benefits c. yes d. yes

(a) circle the verb, (b) underline the subject, (c) determine whether the subject and the verb make sense together, and (d) determine whether the subject agrees in number with the verb. She knows that despite the element of luck, the judgment and the wisdom displayed by each contestant evidently affects the outcome.

a. knows, affect b. she, the judgement and the wisdom c. yes d. no affects -> affects

(a) circle the verb, (b) underline the subject, (c) determine whether the subject and the verb make sense together and (d) determine whether the subject agrees in number with the verb. After all the gardening we did, the sun shining on the flowerbeds make a beautiful sight.

a. makes b. sun c. yes d. no

For each of the underlined modifiers, (a) identify the word or words, if any, that it modifies, and (b) indicate whether the modifier is correct. If the modifier is incorrect, suggest a way to correct the error. A recent formed militia, consisting of *lightly* armed peasants and a few retired army officers, is fighting a *bitterly* civil war against government forces.

a. militia, armed, army officers, civil b. recent -> recently, correct, correct, bitterly -> bitter

Fix the sentence, (a) circle the parallelism markers, and (b) place [square brackets] around each set of parallel elements. The students did poorly on the test more because they had not studied *than the material* was difficult.

a. more, than b. [more BECAUSE they had not studied than BECAUSE the material was difficult]

For each of the underlined modifiers, (a) identify the word or words, if any, that it modifies, and (b) indicate whether the modifier is correct. If the modifier is incorrect, suggest a way to correct the error. The tallest mountain on Earth is Mount Everest *that is on the border between Nepal and Tibet*.

a. mountain b. that -> which

Fix the sentence, (a) circle the parallelism markers, and (b) place [square brackets] around each set of parallel elements. Dr. Crock's claims *have been not corroborated* by other scientists nor published in a prestigious journal but have nonetheless garnered a great deal of attention from the public.

a. neither, nor, but b. [HAVE BEEN neither {CORROBORATED by other scientists nor PUBLISHED in a prestigious journal} but HAVE nonetheless GARNERED a great deal of attention from the public.]

Fix the sentence, (a) circle the parallelism markers, and (b) place [square brackets] around each set of parallel elements. A good night's sleep not only gives your body a chance to rest, *but also energizing you* for the following day.

a. not only, "," but also b. [ not only GIVES your body a chance to rest, but also ENERGIZES you for the following day]

For each of the underlined modifiers, (a) identify the word or words, if any, that it modifies, and (b) indicate whether the modifier is correct. If the modifier is incorrect, suggest a way to correct the error. People, *who talk loudly on their cell phones in crowded trains*, show little respect for other passengers.

a. people b. remove the commas

For each of the underlined modifiers, (a) identify the word or words, if any, that it modifies, and (b) indicate whether the modifier is correct. If the modifier is incorrect, suggest a way to correct the error. People *that are well-informed* know that Bordeaux is a French region *whose most famous export is the wine* *which bears its name*.

a. people, region, wine b. people that are well-informed -> well-informed people, correct, which -> whose most famous wine that bears its name

Fix the sentence, (a) circle the parallelism markers, and (b) place [square brackets] around each set of parallel elements. Many teachers choose to seek employment in the suburbs rather than *facing low salaries* in the city.

a. rather than b. [SEEK employment in the suburbs rather than FACE low salaries in the city]

Underline the idiom, then evaluate as RIGHT, SUSPECT, or WRONG: (a) Advances in the production of high-temperature superconductors are expected to increase the viability of so-called "maglev" trains that float on magnetic fields. (b) It is expected that advances in the production of high-temperature superconductors should increase the viability of so-called "maglev" trains that float on magnetic fields. (c) It is expected that advances in the production of high-temperature superconductors will increase the viability of so-called "maglev" trains that float on magnetic fields. (d) Advances in the production of high-temperature superconductors are expected for them to increase the viability of so-called "maglev" trains that float on magnetic fields.

a. right b. wrong c. right d. wrong

CEO: Over the past several years, we have more than doubled our revenues but profits have steadily declined because an increasing number of customers have failed to pay their balances. In order to compensate for these higher default rates, we will increase the interest charged on outstanding balances from an annual percentage rate (APR) of 9.5% to an APR of 12%. This increase will be sufficient to compensate for the current rate of defaults and allow us to increase our profits. Which of the following statements, if true, would most seriously undermine a plan to increase interest rates in order to spur profitable growth? a. Many other companies have experienced a similar trend in their default rates. b. The company's operating expenses are above the industry average and can be substantially reduced, thus increasing margins. c. The increase in default rates was due to a rise in unemployment, but unemployment rates are expected to drop in the coming months. d. The proposed increase in the APR will, alone, more than double the company's profit margins. e. An increase in the APR charged on credit card balances often results in higher rates of default.

E

Nitrogen triiodide is a highly explosive chemical that is easy to make from only two ingredients: ammonia and concentrated iodine. However, nitrogen triiodide has never been known to be used in a terrorist or criminal attack. Which of the following, if true, is the most likely explanation for the discrepancy described above? a. Ammonia can be bought in a grocery store, but concentrated iodine must be obtained from somewhat more restricted sources, such as chemical supply houses. b. Nitrogen triiodide is only one of several powerful explosives that can be made from ammonia. c. Many terrorists and criminals have used other chemical explosives such as TNT or PETN. d. Airport security devices are typically calibrated to detect nitrogen compounds, such as ammonia and ammonium compounds. e. Nitrogen triiodide is extremely shock sensitive and can detonate as a result of even slight movement.

E

Recently, the tuition at most elite private colleges has been rising faster than inflation. Even before these increases, many low- and middle-income families were unable to afford the full tuition costs for their children at these institutions of higher learning. With the new tuition increases, these colleges will soon cater solely to students with affluent family backgrounds. Which of the following would it be most useful to determine in order to evaluate the argument? a. Whether students from affluent families are more likely to prefer public or private colleges b. Whether students from low- and middle-income families are qualified to attend elite private colleges c. Whether low-income families are less likely to be able to afford tuition costs than middle-income families d. Whether tuition costs at elite public colleges have also been rising faster than inflation e. Whether grants or scholarships are earmarked for students from economically disadvantaged families

E

Sania: The newest workers in the workforce are the most effective digital marketing employees because they are more likely to use social networking websites and tools themselves. Carlos: But effective digital marketing also requires very technical expertise, such as search engine optimization, that is best learned on the job via prolonged exposure and instruction. Carlos responds to Sania by a. demonstrating that Sania's conclusion is based upon evidence that is not relevant to the given situation b. questioning the accuracy of the evidence presented by Sania in support of her conclusion c. reinforcing Sania's argument by contributing an additional piece of evidence in support of her conclusion d. pointing out differences in the qualifications desired by different employers seeking digital marketing employees e. providing an additional piece of evidence that undermines a portion of Sania's claim

E

Traditionally, public school instructors have been compensated according to seniority. Recently, educational experts have criticized the system as one that rewards lackadaisical teaching and reduces motivation to excel. Instead, these experts argue that, to retain exceptional teachers and maintain quality instruction, teachers should receive salaries or bonuses based on performance rather than seniority. Which of the following, if true, most weakens the argument of the educational experts? a. Some teachers express that financial compensation is not the only factor contributing to job satisfaction and teaching performance. b. School districts will develop their own unique compensation structures that may differ greatly from those of other school districts. c. Upon leaving the teaching profession, many young, effective teachers cite a lack of opportunity for more rapid financial advancement as a primary factor in the decision to change careers. d. In school districts that have implemented pay for performance compensation structures, standardized test scores have dramatically increased. e. A merit-based system that bases compensation on teacher performance reduces collaboration, which is an integral component of quality instruction.

E

Two-dimensional barcodes are omni-directional; that is, unlike one-dimensional barcodes, they can be scanned from any direction. Additionally, two-dimensional barcodes are smaller and can store more data than their one-dimensional counterparts. Despite such advantages, two-dimensional barcodes account for a much smaller portion of total barcode usage than one-dimensional barcodes. Which of the following, if true, most helps to resolve the apparent paradox? a. Many smaller stores do not use barcodes at all because of the expense. b. For some products, the amount of data necessary to be coded is small enough to fit fully on a one-dimensional barcode. c. Two-dimensional barcodes are, on average, less expensive than one-dimensional barcodes. d. Two-dimensional barcodes can also be scanned by consumer devices, such as cell phones. e. One-dimensional barcodes last longer and are less prone to error than two-dimensional barcodes.

E

Fix the underlined section: Mozart, who died in 1791, *has lived* in Salzburg for most of his life.

has lived -> lived

Correct the error in the underlined section (if there is one.) If the supplier *has signed* a binding contract, he will deliver the goods.

has signed

Fix the underlined section: By the end of the Apollo program, twelve Americans *have walked* on the moon.

have walked -> had walked

Circle the pronoun, underline the antecedent, and correct the sentence if needed. Jim may not be elected CEO by the board because he does not meet their standards.

he, its their -> its

Correct the error in the underlined section (if there is one.) Ethan is unsure what to do tonight: his boss wants *that he stay* at the office, but his wife insists *that he come* home for dinner.

him to stay--that he come

Fill in the blank space with "like" or "as" Frankie never went to law school, but he believes that years of watching Law & Order have taught him to think _______ a lawyer.

like

Fill in the blank space with "like" or "as" My grandfather eats _______ a child, slurping loudly and helping himself to plenty of ketchup.

like

Fill in the blank space with "like" or "as" The person in the recording sounds _______ a child.

like

Fill in the blank space with "like" or "as" _______ lawyers, doctors are bound by a code of professional ethics.

like

Underline comparison, write correct or correct the sentence: The rapid development of India in the twenty-first century is like England in the eighteenth century.

like like England -> is like that of England

Underline comparison, write correct or correct the sentence: Juggling is a favorite pastime for me, like for you.

like -> as

Underline comparison, write correct or correct the sentence: Law students learn to think like a lawyer does.

like -> as

Circle the pronoun, underline the antecedent, and correct any errors: When the guests finished their soup, they were brought plates of salad.

their, they CORRECT

Circle the pronoun, underline the antecedent, and correct any errors: The players' helmets need to be repainted before they are used in Sunday's game.

they CORRECT

Circle the pronoun, underline the antecedent, and correct any errors: When tetrapods developed lungs, they became the first amphibians capable of surviving on land.

they CORRECT

Underline comparison, write correct or correct the sentence: Courtney's experiences at Haleford, a large research university with renowned professors, affluent students, and imposing buildings, were unlike her high school on the reservation.

unlike experiences...were unlike her high school -> were unlike her experiences

Correct the error in the underlined section (if there is one.) Because Cole *wears* a helmet when he *struck* on the head by a falling coconut ten years ago, he *has escaped* serious injury in that episode.

was wearing, was struck, escaped

Correct the error in the underlined section (if there is one.) Helen would feel better if she *was* my daughter.

were

Fix the underlined section: Water freezes if it *were* cooled to zero degrees Celsius.

were -> is

Fix the underlined section: Last Monday, Mary realized that she *will have* to spend all of that night rewriting her application because she *did not back up* her files.

will have -> would have did not back up -> had not back up

Fix the underlined section: She already *woke up* when the phone rang.

woke up -> woken up

Around 1960, mathematician Edward Lorenz found unexpected behavior in apparently simple equations representing atmospheric air flows. Whenever he reran his model with the same inputs, different outputs resulted, although the model lacked any random elements. Lorenz realized that tiny rounding errors in the initial data mushroomed over time, leading to erratic results. His findings marked a seminal moment in the development of chaos theory, which, despite its name, has little to do with randomness. Lorenz's experiment was one of the first to demonstrate conclusively that unpredictability can arise from deterministic equations, which do not involve chance outcomes. In order to understand this phenomenon, first consider the non-chaotic system of two poppy seeds placed in a round bowl. As the seeds roll to the bowl's center, a position known as a point attractor, the distance between the seeds shrinks. If, instead, the bowl is flipped over, two seeds placed on top will roll away from each other. Such a system, while still not technically chaotic, enlarges initial differences in position. Chaotic systems, such as a machine mixing bread dough, are characterized by both attraction and repulsion. As the dough is stretched, folded, and pressed back together, any poppy seeds sprinkled in are inter-mixed seemingly at random. But this randomness is illusory. In fact, the poppy seeds are captured by "strange attractors," staggeringly complex pathways whose tangles appear accidental but are in fact determined by the system's fundamental equations. During the dough-kneading process, two poppy seeds positioned next to each other eventually go their separate ways. Any early divergence or measurement error is repeatedly amplified by the mixing until the position of any seed becomes effectively unpredictable. It is this "sensitive dependence on initial conditions" and not true randomness that generates unpredictability in chaotic systems, of which one example may be the Earth's weather. According to the popular interpretation of the "Butterfly Effect," a butterfly flapping its wings causes hurricanes. A better understanding is that the butterfly causes uncertainty about the precise state of the air. This microscopic uncertainty grows until it encompasses even hurricanes. Few meteorologists believe that we will ever be able to predict rain or shine for a particular day years in the future. 1. The primary purpose of this passage is to a. explain how non-random systems can produce unpredictable results b. trace the historical development of a scientific theory c. distinguish one theory from its opposite d. describe the spread of a technical model from one field of study to others e. contrast possible causes of weather phenomena 2. According to the passage, what is true about poppy seeds in bread dough, once the dough has been thoroughly mixed? a. They have been individually stretched and folded over like miniature versions of the entire dough. b. They are scattered in random clumps throughout the dough. c. They are accidentally caught in tangled objects called strange attractors. d. They are bound to regularly dispersed patterns of point attractors. e. They are in positions dictated by the underlying equations that govern the mixing process. 3. According to the passage, the small rounding errors in Lorenz's model a. rendered the results unusable for the purposes of scientific research b. were deliberately included to represent tiny fluctuations in atmospheric air currents c. had a surprisingly large impact over time d. were at least partially expected, given the complexity of the actual atmosphere e. shrank to insignificant levels during each trial of the model 4. The passage mentions each of the following as an example or potential example of a chaotic or non-chaotic system EXCEPT a. a dough-mixing machine b. atmospheric weather patterns c. poppy seeds placed on top of an upside-down bowl d. poppy seeds placed in a right-side-up bowl e. fluctuating butterfly flight patterns

1. A 2. E 3. C 4. E

Polygamy in Africa has been a popular topic for social research over the past half-century; it has been analyzed by many distinguished minds and in various well-publicized works. In 1961, when Remi Clignet published his book "Many Wives, Many Powers," he was not alone in his view that in Africa co-wives may be perceived as direct and indirect sources of increased income and prestige. By the 1970s, such arguments had be-come crystallized and popular. Many other African scholars who wrote on the subject became the new champions of this philosophy. For example, in 1983, John Mbiti proclaimed that polygamy is an accepted and respectable institution serving many useful social purposes. Similarly, G.K. Nukunya, in his paper "Polygamy as a Symbol of Status," reiterated Mbiti's idea that a plurality of wives is a legitimate sign of affluence and power in the African society. The colonial missionary voice, however, provided consistent opposition to polygamy. Invoking the authority of the Bible, missionaries argued that the practice was unethical and destructive of family life, and they propagated the view that Africans had to be coerced into abiding by the monogamous view of marriage favored by Western culture. In some instances, missionaries even dictated immediate divorce for newly-converted men who had already entered into polygamous marriages. Unfortunately, neither the missionary voice nor the scholarly voice considered the views of African women important. Although there was some awareness that women regarded polygamy as both a curse and a blessing, the distanced, albeit scientific, perspective of an outside observer predominated both at the pulpit and in scholarly writings. Contemporary research in the social sciences has begun to focus on the protagonist's voice in the study of culture, recognizing that the views and experiences of those who take part in a given reality ought to receive close examination. This privileging of the protagonist seems appropriate, particularly given that women in Africa have often used literary productions to comment on marriage, family, and gender relations. 1. According to the passage, colonial missionaries and popular scholars shared which of the following traits in their approach to the issue of polygamy? a. Both considered polygamy a sign of social status and success. b. Neither accounted for the views of local women. c. Both attempted to limit the prevalence of polygamy. d. Both pointed out polygamy's destructive effects on family life. e. Both exhibited a somewhat negative attitude towards polygamy. 2. The author implies which of the following about Nukunya and Mbiti's works? a. From their point of view, a man who lacks wealth and influence is less likely to have many wives. b. They adjusted their initial views on polygamy, recognizing that the experiences of African women should receive closer attention. c. Their arguments represented a significant departure from those of Remi Clignet. d. Their analyses may have been tainted by the fact that both men practiced polygamy themselves. e. Their views reflected the majority opinion of the African population. 3. The passage mentions each of the following, EXCEPT a. the year of publication of Remi Clignet's book "Many Wives, Many Powers" b. the year in which John Mbiti made a claim that polygamy is an accepted institution c. examples of African women's literary productions devoted to family relations d. reasons for missionary opposition to polygamy e. current-day perspectives with respect to studying polygamy

1. B 2. A 3. C

Historians have long recognized the Japanese sword, or nihonto, as one of the finest cutting weapons ever produced. But to regard the sword that is synonymous with the samurai as merely a weapon is to ignore what makes it so special. The Japanese sword has always been considered a splendid weapon and even a spiritual entity. The traditional Japanese adage "the sword is the soul of the samurai" reflects not only the sword's importance to its wielder but also its permanent connection to its creator, the master smith. Master smiths may not have been considered artists in the classical sense, but each smith exerted great care in the process of creating swords, no two of which were ever forged in exactly the same way. Over hundreds of hours, two types of steel were repeatedly heated, hammered and folded together into thousands of very thin layers, producing a sword with an extremely sharp and durable cutting edge and a flexible, shock-absorbing blade. It was common, though optional, for a master smith to place a physical signature on a blade; moreover, each smith's secret forging techniques left an idiosyncratic structural signature on his blades. Each master smith brought a high level of devotion, skill, and attention to detail to the sword-making process, and the sword it-self was a reflection of his personal honor and ability. This effort made each blade as distinctive as the samurai who wielded it, such that today the Japanese sword is recognized as much for its artistic merit as for its historical significance. 1. Which of the following can be inferred about the structural signature of a Japanese sword? a. It is an inscription that the smith places on the blade during the forging process. b. It refers to the particular characteristics of a blade created by a smith's unique forging process. c. It suggests that each blade can be traced back to a known master smith. d. It reflects the soul of the samurai who wielded the sword. e. It refers to the actual curved shape of the blade. 2. Each of the following is mentioned in the passage EXCEPT a. Every Japanese sword has a unique structure that can be traced back to a special forging process. b. Master smiths kept their forging techniques secret. c. The Japanese sword was considered by some to have a spiritual quality. d. Master smiths are now considered artists by most major historians. e. The Japanese sword is considered both a work of art and a historical artifact. 3. The author explains the way in which swords were made in order to a. establish that the Japanese sword is the most important handheld weapon in history b. claim that the skill of the samurai is what made each Japanese sword unique c. support the contention that the master smiths might be considered artists as well as craftsman d. illustrate that master smiths were more concerned with the artistic merit of their blades than with the blades' practical qualities e. demonstrate that the Japanese sword has more historical importance than artistic importance

1. B 2. D 3. C

In archaeology, as in the physical sciences, new discoveries frequently undermine accepted findings and give rise to new theories. This trend can be seen in the reaction to the recent discovery of a set of 3.3-million-year-old fossils in Ethiopia, the remains of the earliest well-preserved child ever found. The fossilized child was estimated to be about 3 years old at death, female, and a member of the Australopithecus afarensis species. The afarensis species, a major human ancestor, lived in Africa from earlier than 3.7 million to 3 million years ago. "Her completeness, antiquity, and age at death make this find unprecedented in the history of paleoanthropology," said Zeresenay Alemseged, a noted paleoanthropologist. Other scientists said that the discovery could reconfigure conceptions about the lives and capacities of these early humans. Prior to this discovery, it had been thought that the afarensis species had abandoned the arboreal habitat of its ape cousins. However, while the lower limbs of this fossil supported findings that afarensis walked upright, its gorilla-like arms and shoulders suggested that it retained the ability to swing through trees. This has initiated a reexamination of many accepted theories of early human development. Also, the presence of a hyoid bone, a rarely preserved bone in the larynx that supports muscles of the throat, has had a tremendous impact on theories about the origins of speech. The fossil bone is primitive and more similar to that of apes than to that of humans, but it is the first hyoid found in such an early human-related species. 1. The primary purpose of the passage is to a. discuss a controversial scientific discovery b. contrast varying theories of human development c. support a general contention with a specific example d. argue for the importance of a particular field of study e. refute a popular misconception 2. The passage quotes Zeresenay Alemseged in order to a. qualify the main idea of the first paragraph b. provide contrast to the claims of other scientists c. support the theory regarding the linguistic abilities of the afarensis species d. support the stated significance of the discovery e. provide a subjective opinion that is refuted in the second paragraph 3. It can be inferred from the passage's description of the discovery of the fossil hyoid bone that a. Australopithecus afarensis was capable of speech b. the discovered hyoid bone is less primitive than the hyoid bone of apes c. the hyoid bone is necessary for speech d. the discovery of the hyoid bone necessitated the reexamination of prior theories e. the hyoid bone was the most important fossil found at the site 4. Each of the following is cited as a reason that the fossils discovered in Ethiopia were important EXCEPT a. the fact that the remains were those of a child b. the age of the fossils c. the location of the discovery d. the presence of a bone not usually discovered e. the intact nature of the fossils 5. The impact of the discovery of the hyoid bone in the field of archaeology is most closely analogous to which of the following situations? a. The discovery and analysis of cosmic rays lend support to a widely accepted theory of the origin of the universe. b. The original manuscript of a deceased nineteenth-century author confirms ideas about the development of an important work of literature. c. The continued prosperity of a state-run economy stirs debate in the discipline of macroeconomics. d. Newly revealed journal entries by a prominent Civil War—era politician lead to a questioning of certain accepted historical interpretations about the conflict. e. Research into the mapping of the human genome gives rise to nascent applications of individually tailored medicines.

1. C 2. D 3. D 4. C 5. D

Because of the proximity and likeness of Mars to Earth, scientists have long speculated about the possibility of life on Mars. As early as the mid-seventeenth century, astronomers observed polar ice caps on Mars, and by the mid-nineteenth century, scientists discovered other similarities to Earth, including the length of day and axial tilt. But in 1965, photos taken by the Mariner 4 probe revealed a Mars without rivers, oceans or signs of life. Moreover, in the 1990s, it was discovered that Mars, unlike Earth, no longer possessed a substantial global magnetic field, allowing celestial radiation to reach the planet's surface and solar wind to eliminate much of Mars's atmosphere over the course of several billion years. More recent probes have investigated whether there was once liquid water on Mars. Some scientists believe that this question is definitively answered by the presence of certain geological landforms. Others posit that alternative explanations, such as wind erosion or carbon dioxide oceans, may be responsible for these formations. Mars rovers Opportunity and Spirit, which began exploring the surface of Mars in 2004, have both discovered geological evidence of past water activity. In 2013, the rover Curiosity found evidence that the soil on the sur-face of Mars is approximately 2% water by weight. These findings substantially bolster claims that there was once life on Mars. 1. The passage is primarily concerned with which of the following? a. Disproving a widely accepted theory b. Initiating a debate about an unproven theory c. Presenting evidence in support of a recently-formulated claim d. Describing various discoveries made concerning the possibility of life on Mars e. Detailing the findings of the Mars rovers Opportunity, Spirit, and Curiosity 2. Each of the following discoveries is mentioned in the passage EXCEPT: a. Wind erosion and carbon dioxide oceans are responsible for certain geological landforms on Mars. b. Mars does not have a substantial global magnetic field. c. Mars had water activity at some point in the past. d. The length of day on Mars is similar to that on Earth. e. The axial tilt of Mars is similar to that of Earth. 3. The passage suggests which of the following about polar ice caps? a. Until recently, the ones on Mars were thought to consist largely of carbon dioxide. b. The ones on Mars are made almost entirely of frozen water. c. They are also found on Earth. d. Their formation is tied to length of day and axial tilt. e. They indicate that conditions on the planet Mars were once very different than they are at present. 4. It can be inferred from the passage that scientists would be more likely to suspect that Mars once held life if there were evidence of which of the following features? a. Carbon dioxide oceans b. Celestial radiation and solar wind c. High daily level of sunlight reaching the planet's surface d. Volcanic eruptions e. A significant global magnetic field

1. D 2. A 3. C 4. E

Most tennis players strive to strike the ball on the racket's vibration node, more commonly known as the "sweet spot." However, many players are unaware of the existence of a second, lesser-known location on the racket face—the center of percussion—that will also greatly diminish the strain on a player's arm when the ball is struck. In order to understand the physics of this second sweet spot, it is helpful to consider what would happen to a tennis racket if the player's hand were to vanish at the moment of impact with the ball. The impact of the ball would cause the racket to bounce backwards, resulting in a translational motion away from the ball. The tendency of this motion would be to jerk all parts of the racket, including the end of its handle, backward, or away from the ball. Unless the ball happened to hit precisely at the racket's center of mass, the racket would additionally experience a rotational motion around its center of mass—much as a penny that has been struck near its edge will start to spin. Whenever the ball hits the racket face, the effect of this rotational motion is to jerk the end of the handle forward, towards the ball. Depending on where the bail strikes the racket face, one or the other of these motions will predominate. However, there is one point of impact, known as the center of percussion, which causes neither motion to predominate; if a ball strikes this point, the impact does not impart any motion to the end of the handle. The reason for this lack of motion is that the force on the upper part of the hand would be equal and opposite to the force on the lower part of the hand, resulting in no net force on the tennis player's hand or forearm. The center of percussion constitutes a second sweet spot because a tennis player's wrist is typically placed next to the end of the racket's handle. When the player strikes the ball at the center of percussion, her wrist is jerked neither forward nor backward, and she experiences greatly reduced vibration in the arm. The manner in which a tennis player can detect the center of percussion on a given tennis racket follows from the nature of this second sweet spot. The center of percussion can be located via simple trial and error by holding the end of a tennis racket between the finger and thumb and throwing a ball onto the strings. If the handle jumps out of the player's hand, then the ball has missed the center of percussion. 1. The author mentions a penny that has been struck near its edge in order to a. illustrate what happens at the lesser-known center of percussion b. argue that a penny spins in the exact way that a tennis racket spins c. illustrate the difference between two types of motion d. draw an analogy to help explain to help explain a type of motion e. demonstrate that pennies and tennis rackets do not spin in the same way 2. According to the passage, which of the following occurs when a ball strikes the racket strings on a sweet spot? a. The jolt that accompanies most strokes will be more pronounced. b. The racket experiences rotational motion but not translational motion. c. The racket experiences translational motion but not rotational motion. d. The player experiences less vibration in the arm holding the racket. e. The center of mass and the center of percussion coincide. 3. Which of the following can be inferred about the forces acting on the racket handle? a. A player whose grip is anywhere other than at the end of the racket's handle will experience a jolting sensation when striking the ball. b. Striking a ball at the well-known sweet spot will result in fewer vibrations than striking it at the lesser-known sweet spot. c. Striking a ball on the vibration node will impart some amount of motion to the handle of the racket. d. Depending on where the ball strikes, the handle will experience either translational or rotational motion. e. If the player's hand could disappear at the moment of impact, the racket would drop straight to the ground.

1. D 2. D 3. C

Most tennis players strive to strike the ball on the racket's vibration node, more commonly known as the "sweet spot." However, many players are unaware of the existence of a second, lesser-known location on the racket face—the center of percussion—that will also greatly diminish the strain on a player's arm when the ball is struck. In order to understand the physics of this second sweet spot, it is helpful to consider what would happen to a tennis racket if the player's hand were to vanish at the moment of impact with the ball. The impact of the ball would cause the racket to bounce backwards, resulting in a translational motion away from the ball. The tendency of this motion would be to jerk all parts of the racket, including the end of its handle, backward, or away from the ball. Unless the ball happened to hit precisely at the racket's center of mass, the racket would additionally experience a rotational motion around its center of mass—much as a penny that has been struck near its edge will start to spin. Whenever the ball hits the racket face, the effect of this rotational motion is to jerk the end of the handle forward, towards the ball. Depending on where the bail strikes the racket face, one or the other of these motions will predominate. However, there is one point of impact, known as the center of percussion, which causes neither motion to predominate; if a ball strikes this point, the impact does not impart any motion to the end of the handle. The reason for this lack of motion is that the force on the upper part of the hand would be equal and opposite to the force on the lower part of the hand, resulting in no net force on the tennis player's hand or forearm. The center of percussion constitutes a second sweet spot because a tennis player's wrist is typically placed next to the end of the racket's handle. When the player strikes the ball at the center of percussion, her wrist is jerked neither forward nor backward, and she experiences greatly reduced vibration in the arm. The manner in which a tennis player can detect the center of percussion on a given tennis racket follows from the nature of this second sweet spot. The center of percussion can be located via simple trial and error by holding the end of a tennis racket between the finger and thumb and throwing a ball onto the strings. If the handle jumps out of the player's hand, then the ball has missed the center of percussion. 1. What is the primary message the author is trying to convey? a. A proposal for an improvement to the design of tennis rackets b. An examination of the difference between the two types of sweet spot c. A definition of the translational and rotational forces acting on a tennis racket d. A description of the ideal area in which to strike every ball e. An explanation of a lesser-known area on a tennis racket that reduces unwanted vibration 2. What is the primary function served by paragraph two in the context of the entire passage? a. To establish the main idea of the passage b. To provide an explanation of the mechanics of the phenomenon discussed in the passage c. To introduce a counterargument that elucidates the main idea of the passage d. To explain the physics of tennis e. To explain why the main idea of the passage would be useful for tennis

1. E 2. B

Using the underlined modifiers, (1) identify the word that it modifies and (2) indicate whether the modifier is correct. If incorrect fix the error. Kelp is a natural fertilizer *that has become popular among growers of heirloom tomatoes*, *who generally are willing to pay a premium for organic products*.

1. fertilizer, Kelp 2. correct, correct

Historians have long recognized the Japanese sword, or nihonto, as one of the finest cutting weapons ever produced. But to regard the sword that is synonymous with the samurai as merely a weapon is to ignore what makes it so special. The Japanese sword has always been considered a splendid weapon and even a spiritual entity. The traditional Japanese adage "the sword is the soul of the samurai" reflects not only the sword's importance to its wielder but also its permanent connection to its creator, the master smith. Master smiths may not have been considered artists in the classical sense, but each smith exerted great care in the process of creating swords, no two of which were ever forged in exactly the same way. Over hundreds of hours, two types of steel were repeatedly heated, hammered and folded together into thousands of very thin layers, producing a sword with an extremely sharp and durable cutting edge and a flexible, shock-absorbing blade. It was common, though optional, for a master smith to place a physical signature on a blade; moreover, each smith's secret forging techniques left an idiosyncratic structural signature on his blades. Each master smith brought a high level of devotion, skill, and attention to detail to the sword-making process, and the sword it-self was a reflection of his personal honor and ability. This effort made each blade as distinctive as the samurai who wielded it, such that today the Japanese sword is recognized as much for its artistic merit as for its historical significance. 1. The primary purpose of the passage is to a. challenge the observation that the Japanese sword is highly admired by historians b. introduce new information about the forging of Japanese swords c. discuss an obsolete weapon of great historical significance d. argue that Japanese sword makers were motivated by honor e. explain the value attributed to the Japanese sword 2. Which of the following is the primary function of the second paragraph? a. To present an explanation for a change in perception b. To determine the historical significance of Japanese swords c. To discuss the artistic aspects associated with creating Japanese swords d. To compare Japanese master smiths to classical artists e. To review the complete process of making a Japanese sword

1. E 2. C

Polygamy in Africa has been a popular topic for social research over the past half-century; it has been analyzed by many distinguished minds and in various well-publicized works. In 1961, when Remi Clignet published his book "Many Wives, Many Powers," he was not alone in his view that in Africa co-wives may be perceived as direct and indirect sources of increased income and prestige. By the 1970s, such arguments had be-come crystallized and popular. Many other African scholars who wrote on the subject became the new champions of this philosophy. For example, in 1983, John Mbiti proclaimed that polygamy is an accepted and respectable institution serving many useful social purposes. Similarly, G.K. Nukunya, in his paper "Polygamy as a Symbol of Status," reiterated Mbiti's idea that a plurality of wives is a legitimate sign of affluence and power in the African society. The colonial missionary voice, however, provided consistent opposition to polygamy. Invoking the authority of the Bible, missionaries argued that the practice was unethical and destructive of family life, and they propagated the view that Africans had to be coerced into abiding by the monogamous view of marriage favored by Western culture. In some instances, missionaries even dictated immediate divorce for newly-converted men who had already entered into polygamous marriages. Unfortunately, neither the missionary voice nor the scholarly voice considered the views of African women important. Although there was some awareness that women regarded polygamy as both a curse and a blessing, the distanced, albeit scientific, perspective of an outside observer predominated both at the pulpit and in scholarly writings. Contemporary research in the social sciences has begun to focus on the protagonist's voice in the study of culture, recognizing that the views and experiences of those who take part in a given reality ought to receive close examination. This privileging of the protagonist seems appropriate, particularly given that women in Africa have often used literary productions to comment on marriage, family, and gender relations. 1. Which of the following best describes the primary purpose of the passage? a. To discuss scholarly works that view polygamy as a sign of prestige, respect, and affluence in the African society b. To trace the origins of the missionary opposition to African polygamy c. To argue for imposing restrictions on polygamy in African society d. To explore the reasons for women's acceptance of polygamy e. To discuss multiple perspectives on African polygamy and contrast them with contemporary research 2. The third paragraph of the passage plays which of the following roles? a. It discusses the rationale for viewing polygamy as an indication of prestige and affluence in African society. b. It supports the author's view that polygamy is unethical and destructive of family life. c. It contrasts the views of the colonial missionaries with the position of the most recent contemporary research. d. It describes the views on polygamy held by colonial missionaries and indicates a flaw in this vision. e. It demonstrates that the colonial missionaries were ignorant of the scholarly research on polygamy.

1. E 2. D

In the early years of television, Vladimir Zworykin was considered the device's inventor, at least publicly. His loudest champion was his boss, David Sarnoff, then president of RCA and a man regarded even today as "the father of television." Current historians agree, how-ever, that Philo Farnsworth, a self-educated prodigy who was the first to transmit live images, was television's technical inventor. In his own time, Farnsworth's contributions went largely unnoticed, in large part because he was excluded from the process of introducing the invention to a national audience. Sarnoff put televisions into living rooms, and Sarnoff was responsible for a dominant paradigm of the television industry that continues to be relevant today: advertisers pay for the programming so that they can have a receptive audience for their products. Sarnoff had already utilized this construct to develop the radio industry, and it had, within ten years, become ubiquitous. Farnsworth thought the television should be used as an educational tool, but he had little understanding of the business world, and was never able to implement his ideas. Some argue that Sarnoff simply adapted the business model for radio and television from the newspaper industry, replacing the revenue from subscriptions and newsstand purchases with that of television set sales, but Sarnoff promoted himself as nothing less than a visionary. Some television critics argue that the construct Sarnoff implemented has played a negative role in determining the content of the programs themselves, while others contend that it merely created a democratic platform from which the audience can determine the types of programming it desires. 1. The primary purpose of the passage is to a. correct public misconceptions about Farnsworth's role in developing early television programs b. debate the influence of television on popular culture c. challenge the current public perception of Vladimir Zworykin d. chronicle the events that led from the development of radio to the invention of the television e. describe Sarnoff's influence on the public perception of television's inception and the debate around the impact of Sarnoff's paradigm 2. Which of the following best illustrates the relationship between the second and third paragraphs? a. The second paragraph dissects the evolution of a contemporary controversy; the third paragraph presents differing viewpoints on that controversy. b. The second paragraph explores the antithetical intentions of two men involved in the infancy of an industry; the third paragraph details the eventual deterioration of that industry. c. The second paragraph presents differing views of a historical event; the third paragraph represents the author's personal opinion about that event. d. The second paragraph provides details that are necessary to support the author's opinion, which is presented in the third paragraph. e. The second paragraph presents divergent visions about the implementation of a technology; the third paragraph further explores one of those perspectives. 3. According to the passage, the television industry, at its inception, earned revenue from a. advertising only b. advertising and the sale of television sets c. advertising and subscriptions d. subscriptions and the sale of television sets e. advertising, subscriptions, and the sale of television sets 4. The passage suggests that Farnsworth might have earned greater public notoriety for his invention if a. Vladimir Zworykin had been less vocal about his own contributions to the television c. Farnsworth had involved Sarnoff in his plans to develop, manufacture, or distribute the television d. Sarnoff had involved Farnsworth in his plans to develop, manufacture, or distribute the television e. Farnsworth had conducted research into the type of programming the audience most wanted to watch

1. E 2. E 3. B 4. D

Using the underlined modifiers, (1) identify the word that it modifies and (2) indicate whether the modifier is correct. If incorrect fix the error. The acquaintances *who we like most* are those *that flatter us best*.

1. The acquaintances, those 2. who₁ -> whom

Using the underlined modifiers, (1) identify the word that it modifies and (2) indicate whether the modifier is correct. If incorrect fix the error. Some critics *of the Olympian Party candidate Zeus Pater's* are pleased that he has chosen Artemis Rhodes, *the prefect of Alexandria*, to be his running mate; as many commentators have pointed out, her views on social issues are more in line with the Olympian Party platform than are *Pater's*.

1. critics, Artemis Rhodes, views 2. of the Olympian Party candidate Zeus Pater's -> Of the Olympian party candidate Zeus Pater

Using the underlined modifiers, (1) identify the word that it modifies and (2) indicate whether the modifier is correct. If incorrect fix the error. After many years of difficult negotiations, a deal has been reached *that will lower tariffs and end many subsidies*, potentially changing the lives of millions of people in both the developed and the developing world.

1. deal 2. correct

Ethanol, a fuel derived from corn, can be used alone to power cars or along with gasoline to reduce the amount of gas consumed. Unlike gasoline, ethanol is easily renewable since it is primarily converted from the sun's energy. Moreover, com-pared with conventional gasoline, pure ethanol is a cleaner-burning fuel. To save energy and reduce pollution, many individuals advocate the increased usage of ethanol as a primary fuel source in conjunction with or in place of gasoline. In evaluating the recommendation to increase the use of ethanol, it would be important to research all of the following EXCEPT: a. Whether the energy required to grow and process corn used as fuel is greater than the amount of energy ultimately produced b. Whether more energy is saved when using ethanol in conjunction with or in place of gasoline c. Whether ethanol is as efficient a fuel as gasoline d. Whether it is possible to produce more ethanol than is currently produced e. Whether the process of growing corn for fuel would result in as much pollution as does the production of conventional gasoline

A

Government representative: Between 1996 and 2005, the gray wolf population in Minnesota grew nearly 50%; the gray wolf population in Montana increased only 13% during the same period. Clearly, the Minnesota gray wolf population is more likely to survive and thrive long term. Environmentalist: But the gray wolf population in Montana is nearly 8 times the population in Minnesota; above a certain critical breeding number, the population is stable and does not require growth in order to survive. The environmentalist challenges the government representative's argument by doing which of the following? a. introducing evidence that undermines an assumption made by the representative b. challenging the representative's definition of a critical breeding number c. demonstrating that the critical breeding number of the two wolf populations differs significantly d. implying that the two populations of wolves could be combined in order to preserve the species e. suggesting that the Montana wolf population grew at a faster rate than stated in the representative's argument

A

Government restrictions have severely limited the amount of stem cell research U.S. companies can conduct. Because of these restrictions, many U.S. scientists who specialize in the field of stem cell research have signed long-term contracts to work for foreign companies. Recently, Congress has proposed lifting all restrictions on stem cell research. Which of the following statements can most properly be inferred from the information above? a. Some foreign companies that conduct stem cell research work under fewer restrictions than some U.S. companies do. b. Because U.S. scientists are under long-term contracts to foreign companies, there will be a significant influx of foreign professionals into the U.S. c. In all parts of the world, stem cell research is dependent on the financial backing of local government. d. In the near future, U.S. companies will no longer be at the forefront of stem cell research. e. If restrictions on stem cell research are lifted, many of the U.S. scientists will break their contracts to return to U.S. companies.

A

In 2001 the Peruvian government began requiring tourists to buy permits to hike the Inca Trail to the ancient city of Machu Picchu. Only 500 people per day are now allowed to hike the Inca Trail, whereas before 2001 daily visitors numbered in the thousands. The Peruvian government claims that this permit program has successfully prevented deterioration of archaeological treasures along the Inca Trail. Which of the following, if true, most strengthens the argument above? a. Since 2001, lncan ruins similar to Machu Picchu but without a visitor limit have disintegrated at a significantly greater rate than those on the Inca Trail. b. Villages near Machu Picchu have experienced declines in income, as fewer tourists buy fewer craft goods and refreshments. c. Many of the funds from the sale of Inca Trail permits are used to hire guards for archaeological sites without permit programs. d. Since 2001, tourist guides along the Inca Trail have received 50% to 100% increases in take-home pay. e. The total number of tourists in Peru has risen substantially since 2001, even as the number of tourists hiking the Inca Trail has remained constant.

A

In an attempt to explain the cause of malaria, a deadly infectious disease, early European settlers in Hong Kong attributed the malady to poisonous gasses supposedly emanating from low-lying swampland. In the 1880s, however, doctors determined that Anopheles mosquitos were responsible for transmitting the disease to humans after observing that **the female of the species can carry a parasitic protozoan that is passed on to unsuspecting humans when a mosquito feasts on a person's blood.** What function does the statement in boldface fulfill with respect to the argument presented above? a. It provides support for the explanation of a particular phenomenon. b. It presents evidence that contradicts an established fact. c. It offers confirmation of a contested assumption. d. It identifies the cause of an erroneous conclusion. e. It proposes a new conclusion in place of an earlier conjecture.

A

Methyltetrachloride (MTC) is a chemical found in some pesticides, glues, and sealants. Exposure to MTC can cause people to develop asthma. In order to halve the nation's asthma rate, the government plans to ban all products containing MTC. The government's plan to halve the nation's asthma rate relies on which of the following assumptions? a. Exposure to MTC is responsible for no less that half of the nation's asthma cases. b. Products containing MTC are not necessary to the prosperity of the American economy. c. Asthma has reached epidemic proportions. d. Exercise and proper nutrition are helpful in maintaining respiratory health. e. Dust mites and pet dander can also cause asthma.

A

The underlined portion may contain one or more errors. Select (A) if the original version is correct, (B) if the boldface version is correct, (C) if neither is correct, and (D) if both are correct. *No matter how much work it may require*, getting an MBA is an investment that pays off for most people. -Though it doesn't require much work

A

The underlined portion may contain one or more errors. Select (A) if the original version is correct, (B) if the boldface version is correct, (C) if neither is correct, and (D) if both are correct. She is the most dedicated gardener on the block, watering *the more than 50 plants in her yard every day*. -more every day than the 50 plants in her yard

A

Two genes, BRCA1 and BRCA2, are linked to hereditary breast cancer. Therefore, in order to decrease the annual number of mammogram tests administered across a population and to more accurately assess a woman's individual risk of breast cancer, all women should be tested for these genes. Which of the following is an assumption on which the argument depends? a. Some of the women who are tested for the two genes will subsequently undergo mammograms on a less frequent basis than they used to. b. The majority of breast cancer patients have no family history of the disease. c. Researchers may have identified a third breast cancer gene that is linked with hereditary breast cancer. d. Women who have there genes have an 80% chance of getting breast cancer, while women who do not have these genes have only a 10% chance of getting breast cancer. e. The presence of BRCA1 and BRCA2 can explain up to 50% of hereditary cases.

A

Which of the following best completes the passage below? A nonprofit organization in Motor City has proposed that local college students be given the option to buy half-price monthly passes for the city's public transportation system. The nonprofit claims that this plan will reduce air pollution in Motor City while increasing profits for the city's public transportation system. However, this plan is unlikely to meet its goals, since ___________________. a. most college students in Motor City view public transportation as unsafe b. most college students in Motor City view public transportation as prohibitively expensive c. college students typically do not have the 9-to-5 schedules of most workers, and can thus be expected to ride public transportation at times when there are plenty of empty seats d. a bus produces more air pollution per mile than does a car e. a large proportion of the college students in Motor City live off campus

A

Media Critic: Network executives allege that television viewership is decreasing due to the availability of television programs on other platforms, such as the internet and mobile devices. These executives claim that **declining viewership will cause advertising revenue to fall and networks will thus be unable to spend the large sums necessary to produce high-quality programming**. That development, in turn, will lead to a dearth of programming for the very devices that cannibalized television's audience. However, research shows that users of alternative platforms are exposed to new programs, and, **as a result, actually increase the number of hors per eek that they watch television**. This demonstrates that alternative platforms will not prevent networks from increasing revenue. The portions in boldface play which of the following roles in the media critic's argument? a. The first is a trend that weighs against the critic's claim; the second is that claim. b. The first is a prediction that is challenged by the argument; the second is a finding upon which the argument depends. c. The first clarifies the reasoning behind the critic's claim; the second demonstrates why that claim is flawed. d. The first acknowledges a position that the network executives accept as true; the second is a consequence of that position. e. the first opposes the critic's claim through an analogy; the second outlines a scenario in which that claim will not hold.

B

Products with innovative and appealing designs relative to competing products can often command substantially higher prices in the marketplace. **Because design innovations are quickly copied by other manufacturers**, many consumer technology companies charge as much as possible for their new designs to extract as much value as possible from them. But large profits generated by the innovative designs give competitors stronger incentives to copy the designs. Therefore, **the best strategy to maximize overall profit from an innovative new design is to charge less than the greatest possible price.** In the argument above the two portions in boldface play which of the following roles? a. The first is an assumption that supports a described course of action; the second provides a consideration to support a preferred course of action. b. The first is a consideration that helps explain the appeal of a certain strategy; the second presents an alternative strategy endorsed by the argument. c. The first is a phenomenon that makes a specific strategy unlikely to be successful; the second is that strategy. d. The first is a consideration that demonstrates why a particular approach is flawed; the second describes a way to amend that approach. e. The first is a factor used to rationalize a particular strategy; the second is a factor against that strategy.

B

Student Advisor: One of our exchange students faced multiple arguments with her parents over the course of the past year. Not surprisingly, her grade point average (GPA) over the same period showed a steep decline. This is just one example of a general truth: problematic family relationships can cause significant academic difficulties for our students. Which of the following is required for the Student Advisor to claim that problematic family relationships can cause academic difficulties? a. Last year, the exchange student reduced the amount of time spent on academic work, resulting in a lower GPA. b. The decline in the GPA of the exchange student was not the reason for the student's arguments with her parents. c. School GPA is an accurate measure of a student's intellectual ability. d. If proper measures are not taken, the decline in the student's academic performance may become irreversible. e. Fluctuations in academic performance are typical for many students.

B

The Farmsley Film and Performing Arts Center was built three years ago in down-town Metropolis. A recent study shows that, on average, a person who attends a show at the Farmsley Center spends $96 at other downtown businesses on the day of the show. Each of the following, if true, would cast serious doubt on the claim that the Farmsley Center is a significant driver of the economic success of downtown Metropolis EXCEPT: a. People who do not attend a Farmsley Center show spend $103 on average when shopping in the downtown area. b. Restaurants near the Farmsley Center tend to be more expensive than restaurants in other areas of the downtown. c. Most of the people who attend films or performances at the Farmsley Center do so because they are already in the area to shop. d. Tax revenues from all products and services sold in the downtown area have changed little in five years. e. Another downtown theatre is the only one large enough to show popular, newly released films Hollywood films.

B

The redemption rate for e-mailed coupons is far lower than that for traditionally distributed paper coupons. One factor is the "digital divide"—those who might benefit the most from using coupons, such as homemakers, the elderly, and those in low-income households, are less likely to have the knowledge or equipment necessary to go online and receive coupons. Which of the following, if true, does the most to support the claim that the digital divide is responsible for lower electronic coupon redemption rates? a. Computers are available for free in libraries, schools, and community centers. b. The redemption rate of ordinary coupons is particularly high among elderly and low-income people who do not know how to use computers. c. Many homes, including those of elderly and low-income people, do not have high-speed internet connections. d. More homemakers than elderly people would use computers if they had access to them. e. The redemption rate for coupons found on the internet has risen in the last five years.

B

World Bank In 2010, China comprised about 10 percent of the world's gross domestic product (GDP), and its voting share in the World Bank was increased from less than 3% to 4.4%. During the same time frame, France comprised about 4% of the world's GDP and saw its voting share in the World bank drop from 4.3% to 3.8%. Which of the following can be logically concluded from the passage above? a. World Bank voting shares are allocated based upon each country's share of the world's GDP. b. The new ratio of voting share to percentage of world GDP is lower for China than it is for France. c. Gross domestic product is the most important factor in determining voting share at the World Bank. d. China should be upset that its voting share does not match its proportion of the world's GDP. e. France lost some of its voting share to China because China comprised a larger portion of the world's GDP.

B

Identify the role of each sentence or major piece of information: A program instituted by a state government to raise money allows homeowners to prepay their future property taxes at the current rate. Even if the government were to raise the tax rate in a subsequent year, any prepaid taxes would allow the homeowner to maintain taxes at the lower rate, lowering the overall property tax burden over time. For this reason, homeowners should participate in the program.

Background - Premise - Conclusion

Advertising Executive: More than 10 million households now own digital video recorders that can fast-forward over television commercials; approximately 75% of these households fast-forward over at least one commercial per 30-minute program. Because television commercials are not as widely watched as they used to be, they are much less cost-effective today. Which of the following is required in order for the advertising executive to claim that television commercials are less cost-effective today? a. Product placement within television programs is a viable alternative to traditional television commercials. b. The television programs preferred by consumers without digital video recorders are similar to those preferred by consumers with the devices. c. Prior to the advent of digital video recorders, very few television viewers switched channels or left the room when commercials began. d. The cost-effectiveness of television advertising is based less upon how many people watch a particular commercial and more upon the appropriateness of the demographic. e. Many companies find it difficult to determine the exact return on investment for television commercials.

C

As the United States demonstrated during its early development, it is not enough for its citizens simply to have rights; the successful function of a democracy requires that thy also know how to exercise those rights. Access to formal education was one necessary component that helped the U.S. citizenry learn how to exercise its rights. Therefore, in order for a democracy to function successfully, its citizens must have access to a formal education. The author develops the argument by a. using an analogy to establish a precedent for a planned future event b. illustrating differences in the requirements for the functioning of a democracy in question c. introducing an example that illustrates a common principle c. forming a hypothesis that explains apparently contradictory pieces of evidence e. supplying an alternate explanation for a known phenomenon

C

For several years, Nighttime News attracted fewer viewers than World News, which broadcasts its show at the same time as Nighttime News. Recently, the producers of Nighttime News added personal interest stories and increased coverage of sports and weather. The two programs now have a roughly equal number of viewers. Clearly, the recent programming changes persuaded viewers to switch from World News to Nighttime News. The conclusion above is properly drawn if which of the following is assumed? a. Viewers are more interested in sports and weather than in personal interest stories. b. The programming content of Nighttime News is more closely aligned with the interests of the overall audience than is the content of World News. c. Some World News viewers liked the new Nighttime News programming better than they liked the World News programming. d. There are other possible causes for an increase in the number of viewers of Nighttime News, including a recent ad campaign that aired on many local affiliates. e. The quality of World News will remain constant even if Nighttime News improves.

C

In an effort to reduce the number of deer, and therefore decrease the number of automobile accidents caused by deer, the government lengthened the deer hunting season earlier this year. Surprisingly, the number of accidents caused by deer has increased substantially since the introduction of the longer hunting season. All of the following, if true, help to explain the increase in traffic accidents caused by deer EXCEPT: a. The presence of humans in the woods causes the deer to move to new areas, which causes the deer to cross roads more frequently than normal. b. In the area where the deer live, traffic has increased substantially precisely because of the lengthened hunting season. c. Most automobile accidents involving deer result from cars swerving to avoid deer, and leave the deer in question unharmed. d. Deer tend to bolt when hearing gunshots or other loud sounds and are more likely to run across a road without warning. e. A new highway was recently built directly through the state's largest forest, which is the primary habitat of the state's deer population.

C

Museum A will display only undamaged objects of proven authenticity. Doubts have been raised about the origins of a supposedly Mycenaean vase currently on display in the museum's antiquities wing. The only way to establish this vase's authenticity would be to pulverize it, then subject the dust to spectroscopic analysis. The claims above, if true, most strongly support which of the following conclusions? a. Authentic Mycenaean vases are valuable and rare. b. Museum A has been beset with questions about the provenance of many of the items in its antiquities wing. c. The vase in question will no longer be displayed in Museum A. d. Spectroscopic analysis has revolutionized the forensic investigation of art forgery. e. Knowingly or not, many of the world's museums display some forgeries.

C

Studies show that impoverished families give away a larger percentage of their income in charitable donations than do wealthy families. As a result, fundraising consultants recommend that charities direct their marketing efforts toward individuals and families from lower socioeconomic classes in order to maximize the dollar value of incoming donations. Which of the following best explains why the consultants' reasoning is flawed? a. Marketing efforts are only one way to solicit charitable donations. b. Not all impoverished families donate to charity. c. Some charitable marketing efforts are so expensive that the resulting donations fail to cover the costs of the marketing campaign. d. Percentage of income is not necessarily indicative of absolute dollar value. e. People are more likely to donate to the same causes to which their friends donate.

C

The underlined portion may contain one or more errors. Select (A) if the original version is correct, (B) if the boldface version is correct, (C) if neither is correct, and (D) if both are correct. Rising costs of raw materials may result *in fewer employees on the line or product volume*. -could resolve itself in fewer employees on the assembly line or in lower product volume.

C

The underlined portion may contain one or more errors. Select (A) if the original version is correct, (B) if the boldface version is correct, (C) if neither is correct, and (D) if both are correct. The driver picked up the people at *the airport who had been waiting*. -the people who had been waiting at the airport

C

Correct the error in the underlined section (if there is one.) *Having been shown* into the office, Julia waited for the dentist to arrive.

CORRECT

Correct the error in the underlined section (if there is one.) A *frightening* storm *has been lashing* South Padre Island, *forcing* Natalie and Todd to postpone their wedding.

CORRECT

Correct the error in the underlined section (if there is one.) Because epidemiological evidence suggests that some tomatoes *are* contaminated with bacteria, Rachel suggested that Patrick *make* a salad without tomatoes.

CORRECT

Correct the error in the underlined section (if there is one.) Helen would feel better if she *swallowed* this pill.

CORRECT

Correct the error in the underlined section (if there is one.) Louise wanted *to buy* something *to eat*, so she stopped at the ATM *to withdraw* some cash.

CORRECT

CarStore's sales personnel have an average of 15 years' experience selling auto-mobiles, and they regularly sell more cars than other local dealers. Despite this, CarStore has recently implemented a mandatory training program for all sales personnel. Which of the following, if true, best explains the facts given above? a. The sales personnel in CarStore have historically specialized in aggressively selling automobiles and add-on features. b. Salespeople at other local dealers average 10 years' experience. c. It is common for new or less experienced employees to participate in training programs. d. Pricing information, which used to be confidential, has recently been released on the Internet, and many customers try to negotiate lower prices using this data. e. Several retailers that compete directly with CarStore use "customer-centered" sales approaches.

D

Country N's oil production is not sufficient to meet its domestic demand. In order to sharply reduce its dependence on foreign sources of oil, Country N recently embarked on a program requiring all of its automobiles to run on ethanol in addition to gasoline. Combined with its oil production, Country N produces enough ethanol from agricultural by-products to meet its current demand for energy. Which of the following must be assumed in order to conclude that Country N will succeed in its plan to reduce its dependence on foreign oil? a. Electric power is not a superior alternative to ethanol in supplementing automobile gasoline consumption. b. In Country N, domestic production of ethanol is increasing more quickly than domestic oil production. c. Ethanol is suitable for the heating of homes and other applications aside from automobiles. d. In Country N, domestic gasoline consumption is not increasing at a substantially higher rate than domestic oil production. e. Ethanol is as efficient as gasoline in terms of mileage per gallon when used as fuel for automobiles.

D

Editorial: In order to preserve the health of its local economy, Metropolis should not permit a CostMart warehouse department store to open within city limits. It has been demonstrated that when CostMart opens a warehouse department store within a city, the bankruptcy rate of local retailers increases in that city by 20% over the next several years. Which of the following questions would be most useful for evaluating the conclusion of the editorial? a. Does the bankruptcy rate of local retailers in a city generally stabilize several years after a CostMart warehouse department store opens? b. Do most residents of Metropolis currently do almost all of their shopping at stores within the city limits of Metropolis? c. Have other cities that have permitted CostMart warehouse department stores within city limits experienced any economic benefits as a result? d. Is the bankruptcy rate for local retailers in Metropolis higher than in the average city that has permitted a CostMart warehouse department store within city limits? e. Does CostMart plan to hire employees exclusively from within Metropolis for the proposed warehouse department store?

D

Manager: The new manufacturing process should save us time overall, even though the first step of the five-step process will take twice as long as it does under the old process. Under the new process, far fewer of the components will be found defective, and the sole purpose of steps two and three under the old process is to weed out defective components. As a result, we should be able to eliminate two of the five steps in the existing manufacturing process. Which of the following would be most useful in evaluating the claim made in the argument? a. Whether factory workers will require training in order to use the new manufacturing process b. Whether the new process is likely to introduce deficiencies or imperfections that must be corrected c. Whether defective components can be fixed or must be thrown out d. Whether a third manufacturing process would save even more time than both the old and new manufacturing processes e. Whether saving time with the new manufacturing process will ultimately lead to cost savings for the company

D

Many people praise High Renaissance painting for creating very realistic images from observation, **but scholars have documented that some High Renaissance painters used pinhole cameras to project the likeness of their subjects onto the canvas and painted from there** . Thus people who credit High Renaissance painters with superior artistic skills are misguided. **Painting from a projected image requires only an insignificant amount of additional skill beyond that needed to copy a picture outright.** In the argument given, the two boldfaced portions play which of the following roles? a. The first is a finding that has been used to support a conclusion that the argument rejects; the second is a claim that supports that conclusion. b. The first is a finding that has been used to support a conclusion that the argument rejects; the second is that conclusion. c. the first is a claim put forth to support a conclusion that the argument rejects; the second is a consideration that is introduced to counter the force of that evidence. d. The first is evidence that forms the basis for the position that the argument seeks to establish; the second is a claim presented to solidify that position. e. The first is evidence that forms the basis for the position that the argument seeks to establish; the second is that position.

D

Scientists suspect that Europa, a moon orbiting Jupiter, may contain living organisms. However, the government recently scrapped an unmanned science mission to Europa and replaced it with a project aimed at landing an astronaut on Mars. Polls show that the public is far more fascinated by space travel than by discovering life elsewhere in the universe. Critics argue that the government's decision-making process places a greater emphasis on popularity than it does on the importance of scientific research. Which of the following, if true, would most strengthen a contention by the government that the new project is a better use of its funds? a. In the first year of the project, the government will spend 30% of its total budget on developing a space shuttle that can travel to Mars; that figure is expected to drop to 0% after five years. b. The government cannot be absolutely certain of the chances for success of either project. c. Some scientists are convinced that a mission to Europa would add immeasurably to our understanding of the universe. d. hew telescope that has just become available to scientists promises to yield more information than the planned mission to Europa was designed to provide. e. Most people feel that a shuttle to Mars is the next logical step in the development r in the solar of a system that will allow humans to travel even further in the solar system.

D

The Smithtown Theatre, which stages old plays, has announced an expansion that will double its capacity along with its operating costs. The theatre is only slightly profitable at present. In addition, all of the current customers live in Smithtown, and the population of the town is not expected to increase in the next several years. Thus, the expansion of the Smithtown Theatre will prove unprofitable. Which of the following, if true, would most seriously weaken the argument? a. A large movie chain plans to open a new multiplex location in Smithtown later this year. b. Concession sales in the Smithtown Theatre comprise a substantial proportion of the theatre's revenues. c. Many recent arrivals to Smithtown are students who are less likely to attend the Smithtown Theatre than are older residents. d. The expansion would allow the Smithtown Theatre to stage larger, more popular shows that will attract customers from neighboring towns. e. The Board of the Smithtown Theatre often solicits input from residents of the town when choosing which shows to stage.

D

The average fuel efficiency of vehicles sold nationwide during the period 2000-2004 was 25 miles per gallon; the corresponding figure during the period 1995-1999 was 20 miles per gallon. The national average price of gasoline during the period 2000-2004 was $2 per gallon; the corresponding figure during the period 1995-1999 was $1.60 per gallon. The statements above, if true, best support which of the following conclusions? a. The average fuel efficiency of vehicles sold nationwide should reach 30 miles per gallon for the period 2005-2009. b. The national average price of gasoline during 1997 was lower than the corresponding price during 2003. c. Rising gasoline prices led consumers to purchase more fuel-efficient cars. d. Between the two described time periods, the average fuel efficiency of cars sold and the national average gasoline price both increased at roughly the same rate. e. Consumers spent more money on gasoline during the period 2000-2004 than during the period 1995-1999.

D

The owners of a book store and a nearby coffee shop have decided to combine their businesses. Both owners believe that this merger will increase the number of customers and therefore the gross revenue, because customers who come for one reason may also decide to purchase something else. Which of the following, if true, most weakens the owners' conclusion that a merger will increase revenue? a. Books and drinks can both be considered impulse purchases; often, they are purchased by customers without forethought. b. Profit margins at a coffee shop are generally significantly higher than profit margins at a book store. c. People who are able to read the first chapter of a book before buying are more likely to decide to buy the book. d. A large majority of the book store's current customer base already frequents the coffee shop. e. A combination book store and coffee shop that opened in a neighboring city last year has already earned higher than expected profits.

D

The underlined portion may contain one or more errors. Select (A) if the original version is correct, (B) if the boldface version is correct, (C) if neither is correct, and (D) if both are correct. Hector remembers San Francisco *as it was when he left 10 years ago*. -as though time has stood still in the 10 years since he left

D

The underlined portion may contain one or more errors. Select (A) if the original version is correct, (B) if the boldface version is correct, (C) if neither is correct, and (D) if both are correct. Students at Carver High School are encouraged to pursue only those extracurricular activities from which *stems success in college applications*. -success in college applications stems

D

Which of the following most logically completes the argument given below? Deep-brain stimulation is a new technique for combating severe depression. In a recent experiment, electrodes were implanted into the brains of six patients who had not responded to any currently approved treatment for depression. When an electrical current to the electrodes was switched on, four of the patients reported feeling a dramatic reduction in depressive symptoms. The long-term prospects of the new treatment are not promising, however, because _________________. a. other treatments for depression may also be effective b. the other two patients reported only a slight reduction of depressive symptoms during the treatment c. deep-brain stimulation relies on the expertise of highly skilled physicians d. when the electrical current is interrupted, the effects of the treatment are reversed e. in a subsequent experiment, a one-hour treatment with the electrodes resulted in a sustained remission from depression in the four patients for six months

D

Correct the error in the underlined section (if there is one.) Sitting at the kitchen table, *a decision to bake a cake got made by Eric*.

Eric decided to bake a cake

Fix the sentence: Historically, the Isle of Man had an economy based primarily on agriculture and fishing; now, one based on banking, tourism, and film production.

Historically, the Isle of Man had an economy based primarily on agriculture and fishing now, IT HAS one based on banking, tourism, and film production.

Underline comparison, write correct or correct the sentence: Like many other states, Virginia is technically a commonwealth.

Like CORRECT

Circle the pronoun, underline the antecedent, and correct any errors: Meg left all her class notes at school because she decided that she could do her homework without it.

Meg--her, notes--them, she, she, her without it -> without them

Circle the pronoun, underline the antecedent, and correct the sentence if needed. When Norma and her husband read an article about Florida's adorable manatees, they promised each other that they would one day go there and see one.

Norma--her, Norma and her husband--they, each other, they, one go there and see one -> go to Florida and see one

Fix the sentence: Orinoco.com, a major internet retailer, announced mixed results for the second quarter: the number of people shopping at Orinoco.com grew by a 34% rise, but profit per customer fell sharply as consumers shifted to lower-margin items in response to uncertain economic conditions.

Orinoco.com, a major internet retailer, announced mixed results for the second quarter: the number of people shopping at Orinoco.com grew by 34%, but profit per customer fell sharply as consumers shifted to lower-margin items in response to uncertain economic conditions.

Circle the pronoun, underline the antecedent, and correct any errors: Our cat is cuter than those in the shelter.

Our those -> the cats

Identify the role of each sentence or major piece of information: A series of research studies has reported that flaxseed oil can have a beneficial effect in reducing tumor growth in mice, particularly the kind of tumor found in human postmenopausal breast cancer. Thus, flaxseed oil should be recommended as an addition to the diets of all postmenopausal women.

Premise - Conclusion

Identify the role of each sentence or major piece of information: During the past 30 years , the percentage of the population that smokes cigarettes has consistently declined. During the same time period, however, the number of lung cancer deaths attributed to smoking cigarettes has increased.

Premise - Premise

Circle the pronoun, underline the antecedent, and correct any errors: Samantha took her laptop and her books with her on the airplane because she thought she could use these to get some work done.

Samantha--her, her, her, she, she, laptop and books--them these -> them

Circle the pronoun, underline the antecedent, and correct any errors: A few Shakespearean scholars maintain that he borrowed some of his most memorable lines from Christopher Marlowe.

Shakespeare--his replace he with "Shakespeare"

Fix the sentence: The Bentley trench, situated more than a mile and a half below sea level and completely covered by Antarctic glaciers, and it is the lowest point on the planet not under the oceans.

The Bentley trench, situated more than a mile and a half below sea level and completely covered by Antarctic glaciers, IS the lowest point on the plant not under the oceans.

Fix the sentence: It is possible that the earthquake may have caused the building's collapse.

The earthquake may have caused the building's collapse.

Fix the sentence: The municipality's back-to-work program has had notable success nevertheless, it is not suitable for a state-wide rollout for several reasons.

The municipality's back-to-work program has had notable success; nevertheless, it is not suitable for a state-wide rollout for several reasons

Fix the sentence: The music company was afraid of the accelerating decline of sales of compact discs would not be compensated by increased internet revenue.

The music company was afraid THAT the accelerating decline of sales of compact discs would not be compensated by increased internet revenue.

Fix the sentence: The petroleum distillates were so viscous, the engineers had to heat the pipe by nearly 30 degrees.

The petroleum distillates were so viscous THAT the engineers had to heat the pipe by nearly 30 degrees

Underline the idiom, then evaluate as RIGHT, SUSPECT, or WRONG: (a) Faced with the recurrence of natural disasters, such as floods and wildfires, many state governments have imposed significant taxes on their citizens in order to raise funds in advance of the next calamity. (b) Faced with the recurrence of natural disasters, such as floods and wildfires, many state governments have imposed significant taxes on their citizens for raising funds in advance of the next calamity. (c) Faced with the recurrence of natural disasters, such as floods and wildfires, many state governments have imposed significant taxes on their citizens so as to raise funds in advance of the next calamity. (d) Faced with the recurrence of natural disasters, such as floods and wildfires, many state governments have imposed significant taxes on their citizens in order that the governments might raise funds in advance of the next calamity. (e) Faced with the recurrence of natural disasters, such as floods and wildfires, many state governments have imposed significant taxes on their citizens to raise funds in advance of the next calamity.

a. right b. wrong c. wrong d. suspect e. right

Using the corrected sentence, (a) circle the parallelism markers, and (b) place [square brackets] around each set of parallel elements. He received a medal for sinking an enemy ship and the capture of its crew.

a. the b. [the SINKING of an enemy ship and the CAPTURE of its crew]

For each of the underlined modifiers, (a) identify the word or words, if any, that it modifies, and (b) indicate whether the modifier is correct. If the modifier is incorrect, suggest a way to correct the error. *Upon setting foot in the Gothic cathedral*, the *spectacularly* stained-glass windows amazed the *camera-wielding* tourists.

a. tourists b. correct

(a) circle the verb, (b) underline the subject, (c) determine whether the subject and the verb make sense together, and (d) determine whether the subject agrees in number with the verb. Neither she nor her parents understands the challenging math problem.

a. understand b. her parents c. yes d. no understands -> understand

(a) circle the verb, (b) underline the subject, (c) determine whether the subject and the verb make sense together, and (d) determine whether the subject agrees in number with the verb. The young bride, as well as her husband, were amazed by the generosity of the wedding guests.

a. was b. The young bride c. no d. yes were -> was

(a) circle the verb, (b) underline the subject, (c) determine whether the subject and the verb make sense together and (d) determine whether the subject agrees in number with the verb. The decision to place the beautiful artifacts in out-of-the-way nooks around the mansion's various rooms was inspiring.

a. was b. decision c. yes d. yes

(a) circle the verb, (b) underline the subject, (c) determine whether the subject and the verb make sense together and (d) determine whether the subject agrees in number with the verb. A venomous snake designated the emblem of the rebellion by the insurgency.

a. was designated b. snake c. no c. yes

(a) circle the verb, (b) underline the subject, (c) determine whether the subject and the verb make sense together, and (d) determine whether the subject agrees in number with the verb. I was so thirsty that either of the two drinks were fine with me.

a. was, was b. I, either c. yes d. no were -> was

(a) circle the verb, (b) underline the subject, (c) determine whether the subject and the verb make sense together, and (d) determine whether the subject agrees in number with the verb. The traveling salesman was dismayed to learn that neither his sons nor his daughter were interested in moving.

a. was, was b. traveling salesman, his daughter c. yes d. no were -> was

Underline the idiom, then evaluate as RIGHT, SUSPECT, or WRONG: (a) The conflict started both because of ethnic tensions as well as because of economic dislocations. (b) The conflict started both because of ethnic tensions and because of economic dislocations. (c) The conflict started both because of ethnic tensions but also because of economic dislocations.

a. wrong b. right c. wrong

Underline the idiom, then evaluate as RIGHT, SUSPECT, or WRONG: (a) The Caucasus region has several times as many indigenous languages per square mile than most other areas of the world. (b) The Caucasus region has several times as many indigenous languages per square mile as most other areas of the world. (c) The Caucasus region has several times so many indigenous languages per square mile as most other areas of the world. (d) The Caucasus region has several times more indigenous languages per square mile that most other areas of the world. (e) The Caucasus region has several times as many more indigenous languages per square mile as most other areas of the world.

a. wrong b. right c. wrong d. wrong e. wrong

Underline the idiom, then evaluate as RIGHT, SUSPECT, or WRONG: (a) Many places are called Naples—not only the cities in Italy and in Florida, and also a town in Clark County, South Dakota (population 25). (b) Many places are called Naples—not only the cities in Italy and in Florida, but a town in Clark County, South Dakota (population 25) as well. (c) Many places are called Naples—not only the cities in Italy and in Florida, but also a town in Clark County, South Dakota (population 25). (d) Many places are called Naples—the cities in Italy and in Florida, and also a town in Clark County, South Dakota (population 25). (e) Many places are called Naples—not only the cities in Italy and in Florida but a town in Clark County, South Dakota (population 25). (f) Many places are called Naples—not only the cities in Italy and in Florida but, as well, a town in Clark County, South Dakota (population 25).

a. wrong b. suspect c. right d. wrong e. right f. wrong

Underline the idiom, then evaluate as RIGHT, SUSPECT, or WRONG: (a) Unlike humans and guinea pigs, most mammals have the ability of synthesizing Vitamin C from glucose, a simple sugar. (b) Unlike humans and guinea pigs, most mammals have the capability of synthesizing Vitamin C from glucose, a simple sugar. (c) Unlike humans and guinea pigs, most mammals can synthesize Vitamin C from glucose, a simple sugar.

a. wrong b. wrong c. right

Underline the idiom, then evaluate as RIGHT, SUSPECT, or WRONG: (a) These results indicate the health of the marsh's ecosystem has seriously declined. (b) These results are indicative that the health of the marsh's ecosystem has seriously declined. (c) These results indicate that the health of the marsh's ecosystem has seriously declined. (d) These results indicate the health of the marsh's ecosystem to have seriously declined.

a. wrong b. wrong c. right d. wrong

Correct the error in the underlined section (if there is one.) It is necessary that our condominium association *must comply* with the new ordinance, which requires homeowners *clear* the snow from the sidewalks in front of their property.

comply, to clear

Correct the error in the underlined section (if there is one.) If Abraham Lincoln *were born* in Livonia, he *cannot become* the President of the United States.

had been born, could not have become

Correct the error in the underlined section (if there is one.) If the supplier *has signed* a binding contract, he would have delivered the goods.

had signed

Correct the error in the underlined section (if there is one.) When he *swam* across the lake, he lay down on the far shore and relaxed in the sunshine until he was thoroughly dry.

had swum

Underline comparison, write correct or correct the sentence: Hugo is widely acknowledged to be our best employee, because he works harder and more creatively than anyone else in the company.

harder and more creatively than CORRECT

Fix the underlined section: The editor of our local newspaper, who has earned much acclaim in her long career, *has been awarded* a Pulitzer Prize yesterday.

has been awarded -> was awarded

Fix the underlined section: The local government *has built* the school that was destroyed by the earthquake.

has built -> built

Correct the error in the underlined section (if there is one.) Helen may feel better if she *would swallow* this pill.

swallows

Underline comparison, write correct or correct the sentence: Although the towers appear identical, the west tower is the tallest, standing 16 feet taller than the east tower.

taller than the west tower is the tallest, standing 16 feet taller than -> the west tower is 16 feet taller than

Correct the error in the underlined section (if there is one.) *Brokered* by the President of Silonia, the ceasefire agreement mandates *Carpathian forces will cease* their advance into Zapadnia, but allows *them to engage* in limited operations in areas already captured.

that Carpathian forces will cease


Ensembles d'études connexes

10年文法不白學48-was和were的否定句

View Set

PA/AP axial projection: ulnar deviation, scaphoid series, rafert-long method

View Set

Chapter 20: Consumer Choice and Elasticity

View Set

sociology outlines for chapters 3-6

View Set

Module 1c: Cybersecurity Threats, Vulnerabilities and Attacks

View Set